12 svar
508 visningar
anonymousnina 231 – Fd. Medlem
Postad: 25 feb 2018 13:54

Beräkna integraler

Givet är funktionen f(x)=roten ur x. Grafen till denna funktion och linjen x=3 begränsar tillsammans med x-axeln ett område. 

A) Bestäm områdets area. 

Har gjort följande uträkning, 

[2x roten ur x/3], sätter då in x=3, dvs. 2*3 roten ur x/3 = cirka 3,46 

däremot när det kommer till B uppgiften så vet jag inte hur jag ska lösa den, vet att jag måste ställa upp någon funktion eller något dylikt: 

Linjen x=a delar området i två lika stora delar. Bestäm a med tre värdesiffror. 

Har försökt med att dela 3.46 på 2, i och med att det står att a delar området (alltså 3) i två lika stora delar, men svaret blir fel. 

Har försökt med att skriva om a till dess primitiva form, alltså a^2/2, men då vet jag inte hur jag ska gå vidare, eller om man ens överhuvudtaget ska föra så. 

Yngve 37796 – Livehjälpare
Postad: 25 feb 2018 14:19 Redigerad: 25 feb 2018 14:20

EDIT. Missförstod det du skrev.

Yngve 37796 – Livehjälpare
Postad: 25 feb 2018 14:23 Redigerad: 25 feb 2018 14:25

Du har fel integrationsgränser i a-uppgiften. Har du ritat en figur så att du vet hur området ser ut?

---------------------------------------

Integralen i a-uppgiften ska vara 132xx3dx.

I b-uppgiften gäller att hitta ett värde på konstanten a som gör att följande ekvation är uppfylld: 21a2xx3dx=132xx3dx

Guggle 1364
Postad: 25 feb 2018 14:43

Hej Yngve,

Jag tror att uppgiften är f(x)=x f(x)=\sqrt{x} , samt att anonyma nina gjort följande beräkning

03xdx=23x3/203=233.46 \displaystyle \int_0^3 \sqrt{x}\,\mathrm{d}x=\frac{2}{3}\left[ x^{3/2}\right]_0^3 =2\sqrt{3}\approx 3.46

B-uppgiften är alltså att bestämma a så att

0axdx=3 \displaystyle \int_0^a \sqrt{x}\,\mathrm{d}x=\sqrt{3}

anonymousnina 231 – Fd. Medlem
Postad: 25 feb 2018 15:07
Yngve skrev :

Du har fel integrationsgränser i a-uppgiften. Har du ritat en figur så att du vet hur området ser ut?

---------------------------------------

Integralen i a-uppgiften ska vara 132xx3dx.

I b-uppgiften gäller att hitta ett värde på konstanten a som gör att följande ekvation är uppfylld: 21a2xx3dx=132xx3dx

Hur fick du det till 1 i a uppgiften? Försökte läsa av Grafen, men det stod ej några siffror angivet där, så körde bara på 3 och svaret visade sig vara rätt sen. Tror jag även körde på noll också. 

 

Hur kan jag hitta ett värde på konstanten a? Kanske först lösa 1 och se hur mycket det fattas till 3,46? 

Yngve 37796 – Livehjälpare
Postad: 25 feb 2018 16:10

Ja förlåt att jag förvirrade till det hela.

Undre integrationsgränsen ska vara 0 i båda fallen.

anonymousnina 231 – Fd. Medlem
Postad: 25 feb 2018 16:27
Yngve skrev :

Ja förlåt att jag förvirrade till det hela.

Undre integrationsgränsen ska vara 0 i båda fallen.

Det e lugnt :) 

förstår ändå inte hur jag ska ta fram konstanten a? 

Yngve 37796 – Livehjälpare
Postad: 25 feb 2018 16:35
anonymousnina skrev :
Yngve skrev :

Ja förlåt att jag förvirrade till det hela.

Undre integrationsgränsen ska vara 0 i båda fallen.

Det e lugnt :) 

förstår ändå inte hur jag ska ta fram konstanten a? 

Beräkna integralens värde då övre gränsen är a. Kalla detta för I2 I_2 . Detta värde kommer att bero på a.

Om du kallar integralen du beräknade i a-uppgiften för I1 I_1 så är ekvationen du ska lösa I2=I1/2 I_2=I_1/2 .  Dvs integralens värde i b-uppgiften ska vara hälften så stor som den i a-uppgiften.

Detta ger dig en ekvation för värdet på konstanten a.

anonymousnina 231 – Fd. Medlem
Postad: 25 feb 2018 19:48
Yngve skrev :
anonymousnina skrev :
Yngve skrev :

Ja förlåt att jag förvirrade till det hela.

Undre integrationsgränsen ska vara 0 i båda fallen.

Det e lugnt :) 

förstår ändå inte hur jag ska ta fram konstanten a? 

Beräkna integralens värde då övre gränsen är a. Kalla detta för I2 I_2 . Detta värde kommer att bero på a.

Om du kallar integralen du beräknade i a-uppgiften för I1 I_1 så är ekvationen du ska lösa I2=I1/2 I_2=I_1/2 .  Dvs integralens värde i b-uppgiften ska vara hälften så stor som den i a-uppgiften.

Detta ger dig en ekvation för värdet på konstanten a.

Förstår fortfarande inte /:  

Integralens värde, hälften så stor som den i a-uppgiften, dvs 5.19? 

Smaragdalena 78153 – Lärare
Postad: 25 feb 2018 20:49

Integralens värde, hälften så stor som den i a-uppgiften, dvs 5.19?

Hur kan 3,46/2 bli 5,19?

anonymousnina 231 – Fd. Medlem
Postad: 25 feb 2018 21:37
Smaragdalena skrev :

Integralens värde, hälften så stor som den i a-uppgiften, dvs 5.19?

Hur kan 3,46/2 bli 5,19?

Nej det blir ju 1,73. 5.19 räknade jag fram genom att ta 3.46, sedan hälften av det, dvs, 1,73, därefter slå ihop 3.46 med 1.73=5.19. Vet inte hur jag ska gå tillväga med uppgiften, vet att en av integrationsgränserna i B-uppgiften är 0, medans den andra är okänd. 

Smaragdalena 78153 – Lärare
Postad: 25 feb 2018 22:36
Guggle skrev :

Hej Yngve,

Jag tror att uppgiften är f(x)=x f(x)=\sqrt{x} , samt att anonyma nina gjort följande beräkning

03xdx=23x3/203=233.46 \displaystyle \int_0^3 \sqrt{x}\,\mathrm{d}x=\frac{2}{3}\left[ x^{3/2}\right]_0^3 =2\sqrt{3}\approx 3.46

B-uppgiften är alltså att bestämma a så att

0axdx=3 \displaystyle \int_0^a \sqrt{x}\,\mathrm{d}x=\sqrt{3}

Det berättade Guggle för dig för 7 timmar sedan.

Yngve 37796 – Livehjälpare
Postad: 25 feb 2018 22:36 Redigerad: 25 feb 2018 22:37
anonymousnina skrev :Nej det blir ju 1,73. 5.19 räknade jag fram genom att ta 3.46, sedan hälften av det, dvs, 1,73, därefter slå ihop 3.46 med 1.73=5.19. Vet inte hur jag ska gå tillväga med uppgiften, vet att en av integrationsgränserna i B-uppgiften är 0, medans den andra är okänd. 

Nej den är inte okänd.

Du ska beräkna arean av området under grafen från x = 0 till x = a. Den undre integrationsgränsen är alltså 0, den övre integrationsgränsen är a.

Om du beräknar värdet av denna integral så kommer du att få ett uttryck som beror av a. 

Hur ser detta uttryck ut (tips: gör exakt samma beräkning som i a-uppgiften men använd a istället för 3 som övre gräns)?

-----------

Nästa steg:

Detta uttryck ska vara lika med hälften av area du beräknade i a-uppgiften, dvs hälften av 23 2\sqrt{3} . Detta ger dig en ekvation för a.

Svara Avbryt
Close